Une équation avec l’indicateur d’Euler

Bonjour,
Voilà, je suis devant une équation un peu spéciale qui évoque l’indicateur d’Euler, pourriez-vous m’aider à la résoudre, je vous en serai reconnaissante.

Il s’agit de trouver tous les entiers naturels non nuls $a$ et $b$ tels que :
$$2^a + \big(a-\varphi(a)-1\big)! = a^b+1 .
$$ J’ai traité le cas où $a$ est premier, ce qui donne $\varphi(a)=a-1$ donc $2^a=a^b,$ d’où $a=b=2$.
Mais j’ai bloqué dans le cas où $a$ est non premier, j’ai essayé de le décomposer en facteurs premiers ou d’utiliser qu’il admet un diviseur premier, mais toujours rien.

Réponses

  • Il y a une autre solution $a=4$, $b=2$.
  • $a-\phi(a)-1$ compte le nombre d'entiers $x$ compris entre $1$ et $a-1$, tel que $x$ n'est pas premier avec $a$.
    Si $a$ n'est pas premier, $a=pc$ avec $p$ le plus petit premier divisant $a$, et $c>1$, donc $p$ et $2p$ ne sont pas premiers avec $a$. Si $c>2$, $1<p<2p<a$, on a alors $a-\phi(a)-1 \geq 2$.
    (Si $c=2$, alors comme $p\leq c$, $p=2$, et $a=4$, et on retrouve la solution $a=4$, $b=2$)
    Donc si $c>2$, $(a- \phi(a)-1)!$ est divisible par $2$.
    Donc $2^a+(a-\phi(a)-1)!$ est divisible par $2$, or c'est égal à $a^b+1$, donc $a^b$ est égal à $1$ modulo $2$, donc $a$ est impair.
  • Si $a$ est impair et $a=pc$ avec $p$ le plus petit premier divisant $a$ et $c \geq p$. Alors $p,2p, \dots , (c-1)p$ ne sont pas premiers avec $a$ et sont $<$ à $a$, donc $a-\phi(a)-1 \geq c-1$. Donc $(c-1)!$ divise $(a-\phi(a)-1)!$.
    Si $c$ n'est pas premier, alors $c\geq p^2$ et on doit pouvoir montrer que $a$ divise $(c-1)!$ donc $(a-\phi(a)-1)!$.

    Donc $2^a=2^a+(a-\phi(a)-1)!=a^b+1=1 \pmod{a}$. Donc $2^a=1 \pmod{a}$. Donc $2^{pc}=1 \pmod{p}$, donc d'après le petit théorème de Fermat $2^p=2 \pmod{p}$, donc $2^c=1 \pmod{p}$, donc l'ordre de $2$ dans $(\Z/p\Z^*, \times)$ divise $c$ et $p-1$ ( car $2^{p-1}=1 \pmod{p}$ ) , or tous les facteurs premiers de $c$ sont $\geq p$. Donc l'ordre de $2$ est $1$, donc $2=1 \pmod{p}$, ce qui est impossible.

    Reste le cas $a=pq$ avec $p$ et $q$ premier.

    À revoir...
  • Merci beaucoup, mais si a est impair , qu’est ce qu’on peut dire alors?
    J’ai essayé de donner des valeurs à a pour trouver b, mais je trouve toujours que lorsque a n’est pas premier, il n’y a pas de solution, est ce qu’on peut prouver alors que a est forcément pair?
  • Pardon Marco, mais je n’avais pas vu la suite de votre raisonnement.
  • Ma démonstration est à revoir. Peut-être quelqu'un aura trouvé d'ici là...
  • Reste toujours le cas où c n’est pas premier . En tout cas, merci beaucoup.
  • Démonstration plus courte:
    1) Si $a$ est divisible par deux nombres premiers distincts, c'est-à-dire que $a$ ne s'écrit pas $a=p^k$ avec $p$ premier: alors $a=p^kc$ avec $p$ premier, $p \wedge c=1$ et $c>p$.
    $p, 2p, \dots, (p^{k-1}c-1)p$ ne sont pas premiers avec $a$, et sont strictement plus petits que $a$.
    De plus, $c$ aussi n'est pas premier avec $a$ et est strictement plus petit que $a$. Et $c$ n'apparaît pas dans la liste ci-dessus, car il n'est pas divisible par $p$.
    Donc $a-\phi(a)-1 \geq p^{k-1}c-1+1=p^{k-1}c$.
    Donc $(p^{k-1}c)!$ divise $(a-\phi(a)-1)!$.
    $p<c$, donc $p<p^{k-1}c$. Et $p$ et $p^{k-1}c$ apparaissent tous les deux dans la produit $(p^{k-1}c)!$.
    Donc $p^kc=p \times p^{k-1}c$ divise $(p^{k-1}c)!$
    Donc $a=p^k c$ divise $(a-\phi(a)-1)!$.
    Si $2^a+(a-\phi(a)-1)!=a^b+1$, alors $2^a\equiv 1 \pmod a$.
    Donc d'après le lemme ci-dessous, $a=1$, ce qui est impossible.

    Lemme: $2^a\equiv 1 \pmod a$ $\implies a=1$.

    2) Si $a=p^k$ avec $p$ premier, alors $\phi(a)=p^k-p^{k-1}$, donc $a-\phi(a)-1=p^{k-1}-1$.
    a) Si $k>2$, alors $p^{k-1} \geq p^2$, donc $p^{k-1}-1 \geq p^2-1 \geq p$.
    Donc $p$ divise $(p^{k-1} -1)!$, donc $(a-\phi(a)-1)!$. De plus $p$ divise $a^b=p^{kb}$.
    Donc si $ 2^a+(a-\phi(a)-1)!=a^b+1$, alors $2^{p^k} \equiv 1 \pmod p$. Or d'après le petit théorème de Fermat $2^p\equiv 2 \pmod p$, donc $2^{p^k}= (2^p)^{p^{k-1}}\equiv 2^{p^{k-1}}\equiv \cdots \equiv 2 \pmod p$.
    Donc $2\equiv 1 \pmod p$, ce qui est impossible.
    b) Si $k=2$, alors l'équation s'écrit $2^{p^2}+(p-1)!=p^{2b}+1$. (Je ne sais pas si $k=2$).
  • C’est beaucoup plus clair pour moi, et bien détaillé dans le cas 1),
    Dans le cas2) où a=p^k.
    Si k=1 on revient à p premier , c’est facile
    Si k=2, l’équation est équivalente à 2^a+ (p-1)!=p^(2m)+1 , en utilisant le théorème de Wilson, ceci donne que 2^a=2 modulo p.
    Je bloque encore.
  • Je réagis au tout premier message de Marco :

    Si $a$ est premier, l'équation devient $2^a=a^b$ ... qui admet les solutions (2,2) et (4,2)
    Oui.
    Mais 4 n'est pas premier.
    Donc uniquement (2,2)
    Tu me dis, j'oublie. Tu m'enseignes, je me souviens. Tu m'impliques, j'apprends. Benjamin Franklin
  • Je ne suis pas aussi rapide que marco mais je pense avoir trouvé une solution plus courte en distinguant trois cas.

    1) $a=p$ premier : facile.

    2) $a=p^2$ avec $p$ premier : on traite à part $p=2$ et $p=3$ puis pour $p\geq5$ on utilise $p^{2b}\equiv 1\pmod 4$

    3) $a=pc$ avec $p$ plus petit premier divisant $a$ et $c>p$
    On a $a-\varphi(a)\geq p+1$ (les multiples de $c$ ainsi que $p$) donc $p$ divise $(a-\varphi(a)-1)!$ d'où $2^{pc}\equiv 1\pmod p$, d'où $2^c\equiv 1\pmod p$ par le théorème de Fermat.
    L'ordre de $2$ modulo $p$ doit diviser $c$, donc est supérieur ou égal à $p$ ($c$ n'a pas de facteur premier inférieur à $p$) : contradiction avec le théorème de Fermat.
  • Un très grand Merci, c’est clair et très bien expliqué, en tout cas, je n’aurais jamais pu y arriver toute seule.
    Bonne journée.
  • Merci beaucoup Marco, suivre votre raisonnement m’a aidé à m’intéresser de façon plus approfondie et plus courageuse à cet exercice.
  • Lourrran: $a=4$, $b=2$ est solution du problème initial, c'est-à-dire $2^a+(a-\phi(a)-1)!=a^b+1$. En effet, $\phi(4)=2$, et donc on a bien $2^4+(4-2-1)!=4^2+1=17$.
  • Rebonjour, c’est encore moi, une dernière question s’il vous plaît, pourquoi à la fin l’ordre de 2 est >p ? On a c>p mais pourquoi c ne contient pas des facteurs premiers <p ?
    Est ce qu’on ne peut pas avoir par exemple a = pc avec p=7 et c=3.5 =15>p
    Merci.
  • On a supposé que $p$ est le plus petit facteur premier de $a$ donc $c$ ne peut pas avoir un facteur premier strictement plus petit que $p$
  • Tout à fait, j’avais oublié ça.
    Merci beaucoup !!!
Connectez-vous ou Inscrivez-vous pour répondre.